Optimization ( Applied Max and Minimum )

Click For Summary
To construct an open-topped box from a 30 cm square piece of cardboard, squares are cut from each corner, and the volume is maximized by determining the dimensions based on the equation V(x) = x(30-2x)^2. The base of the box is derived as 30 - 2x, reflecting the cuts made on both sides. The volume function simplifies to v(x) = 4x^3 - 120x^2 + 900x, leading to the derivative v'(x) = 12(x-15)(x-5). Critical points are found at x = 5 and x = 15, with x = 5 yielding the largest volume dimensions of 5 cm height and 20 cm base. The second derivative test confirms this as a maximum since v''(5) < 0.
asz304
Messages
107
Reaction score
0

Homework Statement



From a square piece of cardboard, 30 cm on each side, an open topped box is to be constructed by cutting the squares from the corners and turning up the sides. What are the dimensions of the box of largest volume?





The Attempt at a Solution



I know how to do the derivatives from the equation below. but from the book I don't get why the base of the box is : 30 - 2x.

The equation goes like this:
V(x) = x(30-2x)^2.
 
Physics news on Phys.org
base of the box would be 30 - 2x because you are subtracting x twice. Try to draw a diagram to help you

so like you said

v(x) = x(30-2x)(30-2x)
v(x) = 4x^3 - 120x^2 + 900x
v`(x) = 12x^2 - 240x + 900
v`(x) = 12(x-15)(x-5)
v`(x) = 5, 15

Therefore dimestions of largest volume are 5, 20, 20

[note the interval was 0 < x < 15 (if you went higher than 15 you would have negative distance, you can close the intervals or open them depending on how you look at it- but in this case 15 would yield a minimum rather than a maximum)

Check:
v``(x) = 24x - 240
v``(5) = 24(5) < 240
so this is a maximum because v``(x) < 0
 
Question: A clock's minute hand has length 4 and its hour hand has length 3. What is the distance between the tips at the moment when it is increasing most rapidly?(Putnam Exam Question) Answer: Making assumption that both the hands moves at constant angular velocities, the answer is ## \sqrt{7} .## But don't you think this assumption is somewhat doubtful and wrong?

Similar threads

Replies
3
Views
1K
Replies
3
Views
7K
  • · Replies 3 ·
Replies
3
Views
6K
  • · Replies 2 ·
Replies
2
Views
2K
  • · Replies 11 ·
Replies
11
Views
6K
  • · Replies 4 ·
Replies
4
Views
2K
Replies
3
Views
3K
  • · Replies 1 ·
Replies
1
Views
7K
  • · Replies 3 ·
Replies
3
Views
4K
  • · Replies 4 ·
Replies
4
Views
2K